Number theory 13

(Mathematical Reflection Magazine Issue 4- 2013) Giải phương trình nghiệm nguyên dương 10^n-6^m=4n^2.

Lời giải. Nhận thấy m=n=1 là một nghiệm của phương trình.

Thử với n=2,3 thì không thỏa mãn. Với n \ge 4, ta có bổ đề sau:

Bổ đề 1. Nếu a \in \mathbb{N}, \; a \ge 2 thì 10^a>6^a+4a^2.

Bổ đề này dễ chứng minh bằng quy nạp. Do vậy, từ bổ đề ta suy ra 6^m>6^n \Rightarrow m>n \ge 4.

Trường hợp 1. Nếu n lẻ thì 4n^2 \equiv 4 \pmod{16}10^n \equiv 0 \pmod{16} (vì n \ge 4). Do đó 6^m \equiv 12 \pmod{16} \Rightarrow 6^m=16k+12=4(4k+3). Ta suy ra m=2. Tuy nhiên m=2 thì 6^m=6^2 \equiv 4 \pmod{16}, mâu thuẫn.

Trường hợp 2. Nếu n chẵn thì n=2n_1 với n_1 \in \mathbb{N}, \; n_1 \ge 2. Mặt khác, ta có nhận xét sau:

Nhận xét. Nếu a \in \mathbb{Z} thì a^2 \equiv 0,1,2,4 \pmod{7}.

Từ nhận xét ta suy ra 4n^2 \equiv 0,1,2,4 \pmod{7}.

Khả năng 1. Nếu 3|n_1 thì 3|4n^23|6^m nên 3|10^n, mâu thuẫn.

Khả năng 2. Nếu n_1=3k+1 thì theo định lý Fermat nhỏ ta có 10^n=10^{6k+2} \equiv 2 \pmod{7}.

+) Với m chẵn thì 6^m \equiv 1 \pmod{7} \Rightarrow 4n^2 \equiv 1 \pmod{7} (có thể xảy ra).

+) Với m lẻ thì 6^m \equiv 6 \pmod{7} \Rightarrow 4n^2 \equiv 3 \pmod{7}, mâu thuẫn.

Khả năng 3. Nếu n_1=3k+2 thì 10^n=10^{6k+4} \equiv 4 \pmod{7}.

+) Với m chẵn thì 4n^2 \equiv 3 \pmod{7}, mâu thuẫn.

+) Với m lẻ thì 4n^2 \equiv 5 \pmod{7}, mâu thuẫn.

Vậy ba khả năng trên ta suy ra m=2m_1 với m_1 \in \mathbb{N}, \; m_1 \ge 2.

Như vậy (1) \Leftrightarrow \left( 10^{n_1}-6^{m_1} \right) \left( 10^{n_1}+6^{n_1} \right) = 16n_1^2 \qquad (2)

m>n nên (2) \Leftrightarrow 2^{2n_1} \left( 5^{n_1}-2^{m_1-n_1} \cdot 3^{m_1} \right) \left( 5^{n_1}+2^{m_1-n_1} \cdot 3^{m_1} \right)= 16n_1^2

Đặt n_1=2^q \cdot k với q,k \in \mathbb{N},k lẻ. Khi đó phương trình trở thành

\left( 5^{n_1}-2^{m_1-n_1} \cdot 3^{m_1} \right) \left( 5^{n_1}+2^{m_1-n_1} \cdot 3^{m_1} \right)= \dfrac{2^{2q+4}}{2^{2^{q+1} \cdot k}} \cdot k^2

m>n nên các số 5^{n_1}-2^{m_1-n_1} \cdot 3^{m_1}, 5^{n_1}+2^{m_1-n_1} \cdot 3^{m_1} đều là số lẻ. Do đó 2^{2q+4}=2^{2^{q+1} \cdot k} \Rightarrow 2q+4=2^{q+1} \cdot k \Rightarrow q+2=2^q \cdot k.

Ta sẽ đi chứng minh bằng quy nạp: Với mọi q \ge 1,k \ge 3 thì 2^q \cdot k>q+2.

Do đó chỉ có thể k=1. Vậy \left( 5^{n_1}-2^{m_1-n_1} \cdot 3^{m_1} \right) \left( 5^{n_1}+2^{m_1-n_1} \cdot 3^{m_1} \right) =1, điều này hiển nhiên mâu thuẫn do 5^{n_1}+2^{m_1-n_1} \cdot 3^{m_1}>1.

Vậy phương trình có nghiệm nguyên duy nhất \boxed{(m,n)=(1,1)}.

File lời giải: J276Issue 4

  1. Không có bình luận
  1. No trackbacks yet.

Bình luận về bài viết này